Q23

 
amt33
Thanks Received: 0
Vinny Gambini
Vinny Gambini
 
Posts: 2
Joined: December 05th, 2011
 
 
 

Q23

by amt33 Mon Jan 16, 2012 3:24 pm

How do you quickly solve this type of question?
User avatar
 
ManhattanPrepLSAT1
Thanks Received: 1909
Atticus Finch
Atticus Finch
 
Posts: 2851
Joined: October 07th, 2009
 
 
 

Re: Q23

by ManhattanPrepLSAT1 Thu Feb 02, 2012 11:43 am

Good question amt33! These rule substitution questions can sometimes feel like they'll take forever - and sometimes they simply do require trial and error.

On this question, we need to find a rule that will have a similar impact as the original constraint requiring that G begins on the hour. The original constraint restricted G to starting at 1pm, 2pm, or 3pm. Given the way the answer choices are phrased we want to think about not which time G starts but what program it is (remember there are 5 programs total). A 1pm, 2pm, or 3pm start would make G the 1st, 3rd, or 5th program respectively. Given that there are 5 programs, answer choice (C) says this only the other way. Instead of stating when G can start, it says when G cannot - essentially the flip side of the same information.

If coming to that determination isn't something you feel you'll be able to find, then you'll need to use elimination. Go to answer choice (A) and see if you can find one way in which it's different. If so, move to answer choice (B), until you get to one for which you cannot find any differences.

For example...

(A) may give you pause for thought, but this would allow G to begin at 3:30pm while T begins at 1:30pm.
(B) is close but what about 2pm? No reason the original rule precludes G from beginning at 2pm (you may want to run a hypothetical a this point to verify that this works without breaking any rules, which it does). Since G originally could go 3rd, this answer choice does not have the same implications.
(C) would require that G be the 1st, 3rd, or 5th program - which corresponds with an on the hour start.

Hope that helps!
 
samantha.rose.shulman
Thanks Received: 46
Vinny Gambini
Vinny Gambini
 
Posts: 24
Joined: January 16th, 2012
 
 
 

Re: Q23

by samantha.rose.shulman Fri Jul 27, 2012 4:38 pm

Just tagging on here. I've attached a diagrammed solution for Question 23. Hope it helps!
Attachments
PT65, S2, G4, Q23 - Three Hours of TV Programming - ManhattanLSAT.pdf
(76.88 KiB) Downloaded 451 times
 
niksethi12
Thanks Received: 0
Vinny Gambini
Vinny Gambini
 
Posts: 6
Joined: September 12th, 2013
 
 
 

Re: Q23

by niksethi12 Sun Sep 29, 2013 3:56 pm

I have no issues with this problem, but I think I may need a clarifier with the Matt Sherman's explanation.

My general understanding is that (A) does NOT allow the opportunity for G to start at 3:30, but it does allow the opportunity for G to start at 2:30 (per the same explanations given above).

This is because there is only a 3 hour block of programming that these shows must fit into. So if G started at 3:30 it would debase the basic rule stated in the beginning. So it's not that (A) allows 3:30 to be an option (since it was never an option), it's that it allows 2:30 to be an option? Would this thinking be correct?